Derive integer factors of float value? - algorithm

I have a difficult mathematical question that is breaking my brain, my whiteboard, and all my pens. I am working with a file that expresses 2 values, a multiplicand and a percentage. Both of those values must be integers. These two values are multiplied together to produce a range. Range is a float value.
My users edit the range, and I have to calculate a new percentage and multiplicand value. Confused yet? Here's an example:
Multiplicand: 25000 Apples
Percentage: 400 (This works out to .4% or .004)
Range: 100.0 Apples (Calculated by Multiplicand * Percentage)
To complicate things, the allowable values for Percentage are 0-100000. (Meaning 0-100%) Multiplicand is a value between 1 and 32bit int max (presumably unsigned).
I need to allow for users to input a range, like so:
Range: .04 Apples
And calculate the appropriate Percentage and Multiplicand. Using the first example:
OriginalMultiplicand: 25000 Apples
OriginalPercentage: 400 (This works out to .4% or .004)
OriginalRange: 100.0 Apples (Calculated by Multiplicand * Percentage)
NewRange: .01 Apples
NewPercentage: 40
NewMultiplicand: 25 Apples
The example calculation is easy, all that was required was adjusting down the multiplicand and percentage down by the scale factor of the new and old range. The problem arises when the user changes the value to something like 1400.00555. Suddenly I don't have a clean way to adjust the two values.
I need an algorithmic approach to getting values for M & P that produce the closest possible value to the desired range. Any suggestions?

To maximize the numbers of decimal points stored, you should use a P of 1, or 0.1%. If that overflows M, then increment P.
So for your example of 1400.00555, P is 1 and M is 1400006
Your algorithm would search for the lowest P such that M does not overflow. And you can do a binary search here.
public int binarySearch(int P0, int P1) {
P = (P1 - P0)/2;
if(P == P0) {
if(R/(P0/100f) does not overflows 32-bit int) {
return P0;
} else {
return P1;
}
}
if(R/(P/100f) does not overflows 32-bit int) {
return binarySearch(P0, P);
} else {
return binarSearch(P, P1);
}
}
P = binarySearch(1, 100000);
M = round(R/(P/100f));

(I had a bad method here, but I erased it because it sucked.)
EDIT:
There's got to be a better way than that. Let's rephrase the problem:
What you have is an arbitrary floating-point number. You want to represent this floating-point number with two integers. The integers, when multiplied together and then divided by 100000.0, are equal to the floating-point number. The only other constraint is that one of the integers must be equal to or less than 100000.
It's clear that you can't actually represent floating-point numbers accurately. In fact, you can ONLY represent numbers that are expressible in 1/100000s accurately, even if you have an infinite number of digits of precision in "multiplicand". You can represent 333.33333 accurately, with 33333333 as one number and 1 as the other; you just can't get any more 3s.
Given this limitation, I think your best bet is the following:
Multiply your float by 100000 in an integer format, probably a long or some variant of BigNumber.
Factor it. Record all the factors. It doesn't matter if you store them as 2^3 or 2*2*2 or what.
Grab as many factors as you can without the multiplication of them all exceeding 100000. That becomes your percent. (Don't try to do this perfectly; finding the optimal solution is an NP-hard problem.)
Take the rest of the factors and multiply them together. That's your multiplicand.

As I understand from your example, you could represent the range in 100000 different multiplicand * percentage. any choice of multiplicand will give you a satisfying value of percentage, and vice versa. So you have this equation in two variables:
Multiplicand * Percentage = 100.0
You should figure out another equation(constraint), to get a specific value of Multiplicand OR Percentage to solve this equation. Otherwise, you could choose Percentage to be any number between 0-100000 and just substitute it in the first equation to get the value of Multiplicand. I hope I understood the question correctly :)
Edit: OK, then you should factorize the range easily. Get the range, then try to factorize it by dividing range by percentage(2-100000). Once the reminder of division is zero you got the factors. This is a quick pseudo-code:
get range;
percentage = 2;
while(range % percentage != 0)
{
percentage++;
}
multiplicand = range / percentage;
All what you have to do now is to calculate your limits:
max of percentage = 100000;
max of multiplicand = 4294967295;
Max of range = 4294967295 * 100000 = 429496729500000 (15-digit);
your Max range consists of 15 digit at a maximum. double data types in most programming languages can represent it. Do the calculation using doubles and just convert the Multiplicand & Percentage to int at the end.

It seems you want to choose M and P such that R = (M * P) / 100000.
So M * P = 100000 * R, where you have to round the right-hand side to an integer.
I'd multiply the range by 100000, and then choose M and P as factors of the result so that they don't overflow their allowed ranges.

say you have
1) M * P = A
then you have a second value for A, so also new values for M and P, lets call then M2, P2 and A2:
2) M2 * P2 = A2
This I dont know for sure, but that is what you seem to be saying imho: the ratio has to stay the same, then
3) M/P = M2/P2
Now we have 3 equations and 2 unknowns M2 and P2
One way to solve it:
3) becomes
M/P = M2/P2
=>M2 = (M/P)*P2
than substitute that in 2)
(M/P)*P2*P2 = A2
=> P2*P2 = A2 * (P/M)
=> P2 = sqrt(A2 * (P/M))
so first solve P2, then M2 if i didn't make any mistakes
There will have to be some rounding if M2 and P2 have to be integers.
EDIT: i forgot about the integer percentage, so say
P = percentage/100000 or P*100000 = percentage
P2 = percentage2/100000 or P2*100000 = percentage2
so just solve for P2 and M2, and multiply P2 with 100000

Related

Keep uniform distribution after remapping to a new range

Since this is about remapping a uniform distribution to another with a different range, this is not a PHP question specifically although I am using PHP.
I have a cryptographicaly secure random number generator that gives me evenly distributed integers (uniform discrete distribution) between 0 and PHP_INT_MAX.
How do I remap these results to fit into a different range in an efficient manner?
Currently I am using $mappedRandomNumber = $randomNumber % ($range + 1) + $min where $range = $max - $min, but that obvioulsy doesn't work since the first PHP_INT_MAX%$range integers from the range have a higher chance to be picked, breaking the uniformity of the distribution.
Well, having zero knowledge of PHP definitely qualifies me as an expert, so
mentally converting to float U[0,1)
f = r / PHP_MAX_INT
then doing
mapped = min + f*(max - min)
going back to integers
mapped = min + (r * max - r * min)/PHP_MAX_INT
if computation is done via 64bit math, and PHP_MAX_INT being 2^31 it should work
This is what I ended up doing. PRNG 101 (if it does not fit, ignore and generate again). Not very sophisticated, but simple:
public function rand($min = 0, $max = null){
// pow(2,$numBits-1) calculated as (pow(2,$numBits-2)-1) + pow(2,$numBits-2)
// to avoid overflow when $numBits is the number of bits of PHP_INT_MAX
$maxSafe = (int) floor(
((pow(2,8*$this->intByteCount-2)-1) + pow(2,8*$this->intByteCount-2))
/
($max - $min)
) * ($max - $min);
// discards anything above the last interval N * {0 .. max - min -1}
// that fits in {0 .. 2^(intBitCount-1)-1}
do {
$chars = $this->getRandomBytesString($this->intByteCount);
$n = 0;
for ($i=0;$i<$this->intByteCount;$i++) {$n|=(ord($chars[$i])<<(8*($this->intByteCount-$i-1)));}
} while (abs($n)>$maxSafe);
return (abs($n)%($max-$min+1))+$min;
}
Any improvements are welcomed.
(Full code on https://github.com/elcodedocle/cryptosecureprng/blob/master/CryptoSecurePRNG.php)
Here is the sketch how I would do it:
Consider you have uniform random integer distribution in range [A, B) that's what your random number generator provide.
Let L = B - A.
Let P be the highest power of 2 such that P <= L.
Let X be a sample from this range.
First calculate Y = X - A.
If Y >= P, discard it and start with new X until you get an Y that fits.
Now Y contains log2(P) uniformly random bits - zero extend it up to log2(P) bits.
Now we have uniform random bit generator that can be used to provide arbitrary number of random bits as needed.
To generate a number in the target range, let [A_t, B_t) be the target range. Let L_t = B_t - A_t.
Let P_t be the smallest power of 2 such that P_t >= L_t.
Read log2(P_t) random bits and make an integer from it, let's call it X_t.
If X_t >= L_t, discard it and try again until you get a number that fits.
Your random number in the desired range will be L_t + A_t.
Implementation considerations: if your L_t and L are powers of 2, you never have to discard anything. If not, then even in the worst case you should get the right number in less than 2 trials on average.

How to calculate iteratively the running weighted average so that last values to weight most?

I want to implement an iterative algorithm, which calculates weighted average. The specific weight law does not matter, but it should be close to 1 for the newest values and close to 0 to the oldest.
The algorithm should be iterative. i.e. it should not remember all previous values. It should know only one newest value and any aggregative information about past, like previous values of the average, sums, counts etc.
Is it possible?
For example, the following algorithm can be:
void iterate(double value) {
sum *= 0.99;
sum += value;
count++;
avg = sum / count;
}
It will give exponential decreasing weight, which may be not good. Is it possible to have step decreasing weight or something?
EDIT 1
The the requirements for weighing law is follows:
1) The weight decreases into past
2) I has some mean or characteristic duration so that values older this duration matters much lesser than newer ones
3) I should be able to set this duration
EDIT 2
I need the following. Suppose v_i are values, where v_1 is the first. Also suppose w_i are weights. But w_0 is THE LAST.
So, after first value came I have first average
a_1 = v_1 * w_0
After the second value v_2 came, I should have average
a_2 = v_1 * w_1 + v_2 * w_0
With next value I should have
a_3 = v_1 * w_2 + v_2 * w_1 + v_3 * w_0
Note, that weight profile is moving with me, while I am moving along value sequence.
I.e. each value does not have it's own weight all the time. My goal is to have this weight lower while going to past.
First a bit of background. If we were keeping a normal average, it would go like this:
average(a) = 11
average(a,b) = (average(a)+b)/2
average(a,b,c) = (average(a,b)*2 + c)/3
average(a,b,c,d) = (average(a,b,c)*3 + d)/4
As you can see here, this is an "online" algorithm and we only need to keep track of pieces of data: 1) the total numbers in the average, and 2) the average itself. Then we can undivide the average by the total, add in the new number, and divide it by the new total.
Weighted averages are a bit different. It depends on what kind of weighted average. For example if you defined:
weightedAverage(a,wa, b,wb, c,wc, ..., z,wz) = a*wa + b*wb + c*wc + ... + w*wz
or
weightedAverage(elements, weights) = elements·weights
...then you don't need to do anything besides add the new element*weight! If however you defined the weighted average akin to an expected-value from probability:
weightedAverage(elements,weights) = elements·weights / sum(weights)
...then you'd need to keep track of the total weights. Instead of undividing by the total number of elements, you undivide by the total weight, add in the new element&ast;weight, then divide by the new total weight.
Alternatively you don't need to undivide, as demonstrated below: you can merely keep track of the temporary dot product and weight total in a closure or an object, and divide it as you yield (this can help a lot with avoiding numerical inaccuracy from compounded rounding errors).
In python this would be:
def makeAverager():
dotProduct = 0
totalWeight = 0
def averager(newValue, weight):
nonlocal dotProduct,totalWeight
dotProduct += newValue*weight
totalWeight += weight
return dotProduct/totalWeight
return averager
Demo:
>>> averager = makeAverager()
>>> [averager(value,w) for value,w in [(100,0.2), (50,0.5), (100,0.1)]]
[100.0, 64.28571428571429, 68.75]
>>> averager(10,1.1)
34.73684210526316
>>> averager(10,1.1)
25.666666666666668
>>> averager(30,2.0)
27.4
> But my task is to have average recalculated each time new value arrives having old values reweighted. –OP
Your task is almost always impossible, even with exceptionally simple weighting schemes.
You are asking to, with O(1) memory, yield averages with a changing weighting scheme. For example, {values·weights1, (values+[newValue2])·weights2, (values+[newValue2,newValue3])·weights3, ...} as new values are being passed in, for some nearly arbitrarily changing weights sequence. This is impossible due to injectivity. Once you merge the numbers in together, you lose a massive amount of information. For example, even if you had the weight vector, you could not recover the original value vector, or vice versa. There are only two cases I can think of where you could get away with this:
Constant weights such as [2,2,2,...2]: this is equivalent to an on-line averaging algorithm, which you don't want because the old values are not being "reweighted".
The relative weights of previous answers do not change. For example you could do weights of [8,4,2,1], and add in a new element with arbitrary weight like ...+[1], but you must increase all the previous by the same multiplicative factor, like [16,8,4,2]+[1]. Thus at each step, you are adding a new arbitrary weight, and a new arbitrary rescaling of the past, so you have 2 degrees of freedom (only 1 if you need to keep your dot-product normalized). The weight-vectors you'd get would look like:
[w0]
[w0*(s1), w1]
[w0*(s1*s2), w1*(s2), w2]
[w0*(s1*s2*s3), w1*(s2*s3), w2*(s3), w3]
...
Thus any weighting scheme you can make look like that will work (unless you need to keep the thing normalized by the sum of weights, in which case you must then divide the new average by the new sum, which you can calculate by keeping only O(1) memory). Merely multiply the previous average by the new s (which will implicitly distribute over the dot-product into the weights), and tack on the new +w*newValue.
I think you are looking for something like this:
void iterate(double value) {
count++;
weight = max(0, 1 - (count / 1000));
avg = ( avg * total_weight * (count - 1) + weight * value) / (total_weight * (count - 1) + weight)
total_weight += weight;
}
Here I'm assuming you want the weights to sum to 1. As long as you can generate a relative weight without it changing in the future, you can end up with a solution which mimics this behavior.
That is, suppose you defined your weights as a sequence {s_0, s_1, s_2, ..., s_n, ...} and defined the input as sequence {i_0, i_1, i_2, ..., i_n}.
Consider the form: sum(s_0*i_0 + s_1*i_1 + s_2*i_2 + ... + s_n*i_n) / sum(s_0 + s_1 + s_2 + ... + s_n). Note that it is trivially possible to compute this incrementally with a couple of aggregation counters:
int counter = 0;
double numerator = 0;
double denominator = 0;
void addValue(double val)
{
double weight = calculateWeightFromCounter(counter);
numerator += weight * val;
denominator += weight;
}
double getAverage()
{
if (denominator == 0.0) return 0.0;
return numerator / denominator;
}
Of course, calculateWeightFromCounter() in this case shouldn't generate weights that sum to one -- the trick here is that we average by dividing by the sum of the weights so that in the end, the weights virtually seem to sum to one.
The real trick is how you do calculateWeightFromCounter(). You could simply return the counter itself, for example, however note that the last weighted number would not be near the sum of the counters necessarily, so you may not end up with the exact properties you want. (It's hard to say since, as mentioned, you've left a fairly open problem.)
This is too long to post in a comment, but it may be useful to know.
Suppose you have:
w_0*v_n + ... w_n*v_0 (we'll call this w[0..n]*v[n..0] for short)
Then the next step is:
w_0*v_n1 + ... w_n1*v_0 (and this is w[0..n1]*v[n1..0] for short)
This means we need a way to calculate w[1..n1]*v[n..0] from w[0..n]*v[n..0].
It's certainly possible that v[n..0] is 0, ..., 0, z, 0, ..., 0 where z is at some location x.
If we don't have any 'extra' storage, then f(z*w(x))=z*w(x + 1) where w(x) is the weight for location x.
Rearranging the equation, w(x + 1) = f(z*w(x))/z. Well, w(x + 1) better be constant for a constant x, so f(z*w(x))/z better be constant. Hence, f must let z propagate -- that is, f(z*w(x)) = z*f(w(x)).
But here again we have an issue. Note that if z (which could be any number) can propagate through f, then w(x) certainly can. So f(z*w(x)) = w(x)*f(z). Thus f(w(x)) = w(x)/f(z).
But for a constant x, w(x) is constant, and thus f(w(x)) better be constant, too. w(x) is constant, so f(z) better be constant so that w(x)/f(z) is constant. Thus f(w(x)) = w(x)/c where c is a constant.
So, f(x)=c*x where c is a constant when x is a weight value.
So w(x+1) = c*w(x).
That is, each weight is a multiple of the previous. Thus, the weights take the form w(x)=m*b^x.
Note that this assumes the only information f has is the last aggregated value. Note that at some point you will be reduced to this case unless you're willing to store a non-constant amount of data representing your input. You cannot represent an infinite length vector of real numbers with a real number, but you can approximate them somehow in a constant, finite amount of storage. But this would merely be an approximation.
Although I haven't rigorously proven it, it is my conclusion that what you want is impossible to do with a high degree of precision, but you may be able to use log(n) space (which may as well be O(1) for many practical applications) to generate a quality approximation. You may be able to use even less.
I tried to practically code something (in Java). As has been said, your goal is not achievable. You can only count average from some number of last remembered values. If you don't need to be exact, you can approximate the older values. I tried to do it by remembering last 5 values exactly and older values only SUMmed by 5 values, remembering the last 5 SUMs. Then, the complexity is O(2n) for remembering last n+n*n values. This is a very rough approximation.
You can modify the "lastValues" and "lasAggregatedSums" array sizes as you want. See this ascii-art picture trying to display a graph of last values, showing that the first columns (older data) are remembered as aggregated value (not individually), and only the earliest 5 values are remembered individually.
values:
#####
##### ##### #
##### ##### ##### # #
##### ##### ##### ##### ## ##
##### ##### ##### ##### ##### #####
time: --->
Challenge 1: My example doesn't count weights, but I think it shouldn't be problem for you to add weights for the "lastAggregatedSums" appropriately - the only problem is, that if you want lower weights for older values, it would be harder, because the array is rotating, so it is not straightforward to know which weight for which array member. Maybe you can modify the algorithm to always "shift" values in the array instead of rotating? Then adding weights shouldn't be a problem.
Challenge 2: The arrays are initialized with 0 values, and those values are counting to the average from the beginning, even when we haven't receive enough values. If you are running the algorithm for long time, you probably don't bother that it is learning for some time at the beginning. If you do, you can post a modification ;-)
public class AverageCounter {
private float[] lastValues = new float[5];
private float[] lastAggregatedSums = new float[5];
private int valIdx = 0;
private int aggValIdx = 0;
private float avg;
public void add(float value) {
lastValues[valIdx++] = value;
if(valIdx == lastValues.length) {
// count average of last values and save into the aggregated array.
float sum = 0;
for(float v: lastValues) {sum += v;}
lastAggregatedSums[aggValIdx++] = sum;
if(aggValIdx >= lastAggregatedSums.length) {
// rotate aggregated values index
aggValIdx = 0;
}
valIdx = 0;
}
float sum = 0;
for(float v: lastValues) {sum += v;}
for(float v: lastAggregatedSums) {sum += v;}
avg = sum / (lastValues.length + lastAggregatedSums.length * lastValues.length);
}
public float getAvg() {
return avg;
}
}
you can combine (weighted sum) exponential means with different effective window sizes (N) in order to get the desired weights.
Use more exponential means to define your weight profile more detailed.
(more exponential means also means to store and calculate more values, so here is the trade off)
A memoryless solution is to calculate the new average from a weighted combination of the previous average and the new value:
average = (1 - P) * average + P * value
where P is an empirical constant, 0 <= P <= 1
expanding gives:
average = sum i (weight[i] * value[i])
where value[0] is the newest value, and
weight[i] = P * (1 - P) ^ i
When P is low, historical values are given higher weighting.
The closer P gets to 1, the more quickly it converges to newer values.
When P = 1, it's a regular assignment and ignores previous values.
If you want to maximise the contribution of value[N], maximize
weight[N] = P * (1 - P) ^ N
where 0 <= P <= 1
I discovered weight[N] is maximized when
P = 1 / (N + 1)

Split a number into three buckets with constraints

Is there a good algorithm to split a randomly generated number into three buckets, each with constraints as to how much of the total they may contain.
For example, say my randomly generated number is 1,000 and I need to split it into buckets a, b, and c.
These ranges are only an example. See my edit for possible ranges.
Bucket a may only be between 10% - 70% of the number (100 - 700)
Bucket b may only be between 10% - 50% of the number (100 - 500)
Bucket c may only be between 5% - 25% of the number (50 - 250)
a + b + c must equal the randomly generated number
You want the amounts assigned to be completely random so there's just as equal a chance of bucket a hitting its max as bucket c in addition to as equal a chance of all three buckets being around their percentage mean.
EDIT: The following will most likely always be true: low end of a + b + c < 100%, high end of a + b + c > 100%. These percentages are only to indicate acceptable values of a, b, and c. In a case where a is 10% while b and c are their max (50% and 25% respectively) the numbers would have to be reassigned since the total would not equal 100%. This is the exact case I'm trying to avoid by finding a way to assign these numbers in one pass.
I'd like to find a way to pick these number randomly within their range in one pass.
The problem is equivalent to selecting a random point in an N-dimensional object (in your example N=3), the object being defined by the equations (in your example):
0.1 <= x <= 0.7
0.1 <= y <= 0.5
0.05 <= z <= 0.25
x + y + z = 1 (*)
Clearly because of the last equation (*) one of the coordinates is redundant, i.e. picking values for x and y dictates z.
Eliminating (*) and one of the other equations leaves us with an (N-1)-dimensional box, e.g.
0.1 <= x <= 0.7
0.1 <= y <= 0.5
that is cut by the inequality
0.05 <= (1 - x - y) <= 0.25 (**)
that derives from (*) and the equation for z. This is basically a diagonal stripe through the box.
In order for the results to be uniform, I would just repeatedly sample the (N-1)-dimensional box, and accept the first sampled point that fulfills (**). Single-pass solutions might end up having biased distributions.
Update: Yes, you're right, the result is not uniformly distributed.
Let's say your percent values are natural numbers (if this assumption is wrong, you don't have to read further :) In that case I don't have a solution).
Let's define an event e as a tuple of 3 values (percentage of each bucket): e = (pa, pb, pc). Next, create all possible events en. What you have here is a tuple space consisting of a discrete number of events. All of the possible events should have the same possibility to occur.
Let's say we have a function f(n) => en. Then, all we have to do is take a random number n and return en in a single pass.
Now, the problem remains to create such a function f :)
In pseudo code, a very slow method (just for illustration):
function f(n) {
int c = 0
for i in [10..70] {
for j in [10..50] {
for k in [5..25] {
if(i + j + k == 100) {
if(n == c) {
return (i, j, k) // found event!
} else {
c = c + 1
}
}
}
}
}
}
What you have know is a single pass solution, but problem is only moved away. The function f is very slow. But you can do better: I think you can calculate everything a bit faster if you set your ranges correctly and calculate offsets instead of iterating through your ranges.
Is this clear enough?
First of all you probably have to adjust your ranges. 10% in bucket a is not possible, since you can't get condition a+b+c = number to hold.
Concerning your question: (1) Pick a random number for bucket a inside your range, then (2) update the range for bucket b with minimum and maximum percentage (you should only narrow the range). Then (3) pick a random number for bucket b. In the end c should be calculated that your condition holds (4).
Example:
n = 1000
(1) a = 40%
(2) range b [35,50], because 40+35+25 = 100%
(3) b = 45%
(4) c = 100-40-45 = 15%
Or:
n = 1000
(1) a = 70%
(2) range b [10,25], because 70+25+5 = 100%
(3) b = 20%
(4) c = 100-70-20 = 10%
It is to check whether all the events are uniformly distributed. If that should be a problem you might want to randomize the range update in step 2.

How to generate a number in arbitrary range using random()={0..1} preserving uniformness and density?

Generate a random number in range [x..y] where x and y are any arbitrary floating point numbers. Use function random(), which returns a random floating point number in range [0..1] from P uniformly distributed numbers (call it "density"). Uniform distribution must be preserved and P must be scaled as well.
I think, there is no easy solution for such problem. To simplify it a bit, I ask you how to generate a number in interval [-0.5 .. 0.5], then in [0 .. 2], then in [-2 .. 0], preserving uniformness and density? Thus, for [0 .. 2] it must generate a random number from P*2 uniformly distributed numbers.
The obvious simple solution random() * (x - y) + y will generate not all possible numbers because of the lower density for all abs(x-y)>1.0 cases. Many possible values will be missed. Remember, that random() returns only a number from P possible numbers. Then, if you multiply such number by Q, it will give you only one of P possible values, scaled by Q, but you have to scale density P by Q as well.
If I understand you problem well, I will provide you a solution: but I would exclude 1, from the range.
N = numbers_in_your_random // [0, 0.2, 0.4, 0.6, 0.8] will be 5
// This turns your random number generator to return integer values between [0..N[;
function randomInt()
{
return random()*N;
}
// This turns the integer random number generator to return arbitrary
// integer
function getRandomInt(maxValue)
{
if (maxValue < N)
{
return randomInt() % maxValue;
}
else
{
baseValue = randomInt();
bRate = maxValue DIV N;
bMod = maxValue % N;
if (baseValue < bMod)
{
bRate++;
}
return N*getRandomInt(bRate) + baseValue;
}
}
// This will return random number in range [lower, upper[ with the same density as random()
function extendedRandom(lower, upper)
{
diff = upper - lower;
ndiff = diff * N;
baseValue = getRandomInt(ndiff);
baseValue/=N;
return lower + baseValue;
}
If you really want to generate all possible floating point numbers in a given range with uniform numeric density, you need to take into account the floating point format. For each possible value of your binary exponent, you have a different numeric density of codes. A direct generation method will need to deal with this explicitly, and an indirect generation method will still need to take it into account. I will develop a direct method; for the sake of simplicity, the following refers exclusively to IEEE 754 single-precision (32-bit) floating point numbers.
The most difficult case is any interval that includes zero. In that case, to produce an exactly even distribution, you will need to handle every exponent down to the lowest, plus denormalized numbers. As a special case, you will need to split zero into two cases, +0 and -0.
In addition, if you are paying such close attention to the result, you will need to make sure that you are using a good pseudorandom number generator with a large enough state space that you can expect it to hit every value with near-uniform probability. This disqualifies the C/Unix rand() and possibly the*rand48() library functions; you should use something like the Mersenne Twister instead.
The key is to dissect the target interval into subintervals, each of which is covered by different combination of binary exponent and sign: within each subinterval, floating point codes are uniformly distributed.
The first step is to select the appropriate subinterval, with probability proportional to its size. If the interval contains 0, or otherwise covers a large dynamic range, this may potentially require a number of random bits up to the full range of the available exponent.
In particular, for a 32-bit IEEE-754 number, there are 256 possible exponent values. Each exponent governs a range which is half the size of the next greater exponent, except for the denormalized case, which is the same size as the smallest normal exponent region. Zero can be considered the smallest denormalized number; as mentioned above, if the target interval straddles zero, the probability of each of +0 and -0 should perhaps be cut in half, to avoid doubling its weight.
If the subinterval chosen covers the entire region governed by a particular exponent, all that is necessary is to fill the mantissa with random bits (23 bits, for 32-bit IEEE-754 floats). However, if the subinterval does not cover the entire region, you will need to generate a random mantissa that covers only that subinterval.
The simplest way to handle both the initial and secondary random steps may be to round the target interval out to include the entirety of all exponent regions partially covered, then reject and retry numbers that fall outside it. This allows the exponent to be generated with simple power-of-2 probabilities (e.g., by counting the number of leading zeroes in your random bitstream), as well as providing a simple and accurate way of generating a mantissa that covers only part of an exponent interval. (This is also a good way of handling the +/-0 special case.)
As another special case: to avoid inefficient generation for target intervals which are much smaller than the exponent regions they reside in, the "obvious simple" solution will in fact generate fairly uniform numbers for such intervals. If you want exactly uniform distributions, you can generate the sub-interval mantissa by using only enough random bits to cover that sub-interval, while still using the aforementioned rejection method to eliminate values outside the target interval.
well, [0..1] * 2 == [0..2] (still uniform)
[0..1] - 0.5 == [-0.5..0.5] etc.
I wonder where have you experienced such an interview?
Update: well, if we want to start caring about losing precision on multiplication (which is weird, because somehow you did not care about that in the original task, and pretend we care about "number of values", we can start iterating. In order to do that, we need one more function, which would return uniformly distributed random values in [0..1) — which can be done by dropping the 1.0 value would it ever appear. After that, we can slice the whole range in equal parts small enough to not care about losing precision, choose one randomly (we have enough randomness to do that), and choose a number in this bucket using [0..1) function for all parts but the last one.
Or, you can come up with a way to code enough values to care about—and just generate random bits for this code, in which case you don't really care whether it's [0..1] or just {0, 1}.
Let me rephrase your question:
Let random() be a random number generator with a discrete uniform distribution over [0,1). Let D be the number of possible values returned by random(), each of which is precisely 1/D greater than the previous. Create a random number generator rand(L, U) with a discrete uniform distribution over [L, U) such that each possible value is precisely 1/D greater than the previous.
--
A couple quick notes.
The problem in this form, and as you phrased it is unsolvable. That
is, if N = 1 there is nothing we can do.
I don't require that 0.0 be one of the possible values for random(). If it is not, then it is possible that the solution below will fail when U - L < 1 / D. I'm not particularly worried about that case.
I use all half-open ranges because it makes the analysis simpler. Using your closed ranges would be simple, but tedious.
Finally, the good stuff. The key insight here is that the density can be maintained by independently selecting the whole and fractional parts of the result.
First, note that given random() it is trivial to create randomBit(). That is,
randomBit() { return random() >= 0.5; }
Then, if we want to select one of {0, 1, 2, ..., 2^N - 1} uniformly at random, that is simple using randomBit(), just generate each of the bits. Call this random2(N).
Using random2() we can select one of {0, 1, 2, ..., N - 1}:
randomInt(N) { while ((val = random2(ceil(log2(N)))) >= N); return val; }
Now, if D is known, then the problem is trivial as we can reduce it to simply choosing one of floor((U - L) * D) values uniformly at random and we can do that with randomInt().
So, let's assume that D is not known. Now, let's first make a function to generate random values in the range [0, 2^N) with the proper density. This is simple.
rand2D(N) { return random2(N) + random(); }
rand2D() is where we require that the difference between consecutive possible values for random() be precisely 1/D. If not, the possible values here would not have uniform density.
Next, we need a function that selects a value in the range [0, V) with the proper density. This is similar to randomInt() above.
randD(V) { while ((val = rand2D(ceil(log2(V)))) >= V); return val; }
And finally...
rand(L, U) { return L + randD(U - L); }
We now may have offset the discrete positions if L / D is not an integer, but that is unimportant.
--
A last note, you may have noticed that several of these functions may never terminate. That is essentially a requirement. For example, random() may have only a single bit of randomness. If I then ask you to select from one of three values, you cannot do so uniformly at random with a function that is guaranteed to terminate.
Consider this approach:
I'm assuming the base random number generator in the range [0..1]
generates among the numbers
0, 1/(p-1), 2/(p-1), ..., (p-2)/(p-1), (p-1)/(p-1)
If the target interval length is less than or equal to 1,
return random()*(y-x) + x.
Else, map each number r from the base RNG to an interval in the
target range:
[r*(p-1)*(y-x)/p, (r+1/(p-1))*(p-1)*(y-x)/p]
(i.e. for each of the P numbers assign one of P intervals with length (y-x)/p)
Then recursively generate another random number in that interval and
add it to the interval begin.
Pseudocode:
const p;
function rand(x, y)
r = random()
if y-x <= 1
return x + r*(y-x)
else
low = r*(p-1)*(y-x)/p
high = low + (y-x)/p
return x + low + rand(low, high)
In real math: the solution is just the provided:
return random() * (upper - lower) + lower
The problem is that, even when you have floating point numbers, only have a certain resolution. So what you can do is apply above function and add another random() value scaled to the missing part.
If I make a practical example it becomes clear what I mean:
E.g. take random() return value from 0..1 with 2 digits accuracy, ie 0.XY, and lower with 100 and upper with 1100.
So with above algorithm you get as result 0.XY * (1100-100) + 100 = XY0.0 + 100.
You will never see 201 as result, as the final digit has to be 0.
Solution here would be to generate again a random value and add it *10, so you have accuracy of one digit (here you have to take care that you dont exceed your given range, which can happen, in this case you have to discard the result and generate a new number).
Maybe you have to repeat it, how often depends on how many places the random() function delivers and how much you expect in your final result.
In a standard IEEE format has a limited precision (i.e. double 53 bits). So when you generate a number this way, you never need to generate more than one additional number.
But you have to be careful that when you add the new number, you dont exceed your given upper limit. There are multiple solutions to it: First if you exceed your limit, you start from new, generating a new number (dont cut off or similar, as this changes the distribution).
Second possibility is to check the the intervall size of the missing lower bit range, and
find the middle value, and generate an appropiate value, that guarantees that the result will fit.
You have to consider the amount of entropy that comes from each call to your RNG. Here is some C# code I just wrote that demonstrates how you can accumulate entropy from low-entropy source(s) and end up with a high-entropy random value.
using System;
using System.Collections.Generic;
using System.Security.Cryptography;
namespace SO_8019589
{
class LowEntropyRandom
{
public readonly double EffectiveEntropyBits;
public readonly int PossibleOutcomeCount;
private readonly double interval;
private readonly Random random = new Random();
public LowEntropyRandom(int possibleOutcomeCount)
{
PossibleOutcomeCount = possibleOutcomeCount;
EffectiveEntropyBits = Math.Log(PossibleOutcomeCount, 2);
interval = 1.0 / PossibleOutcomeCount;
}
public LowEntropyRandom(int possibleOutcomeCount, int seed)
: this(possibleOutcomeCount)
{
random = new Random(seed);
}
public int Next()
{
return random.Next(PossibleOutcomeCount);
}
public double NextDouble()
{
return interval * Next();
}
}
class EntropyAccumulator
{
private List<byte> currentEntropy = new List<byte>();
public double CurrentEntropyBits { get; private set; }
public void Clear()
{
currentEntropy.Clear();
CurrentEntropyBits = 0;
}
public void Add(byte[] entropy, double effectiveBits)
{
currentEntropy.AddRange(entropy);
CurrentEntropyBits += effectiveBits;
}
public byte[] GetBytes(int count)
{
using (var hasher = new SHA512Managed())
{
count = Math.Min(count, hasher.HashSize / 8);
var bytes = new byte[count];
var hash = hasher.ComputeHash(currentEntropy.ToArray());
Array.Copy(hash, bytes, count);
return bytes;
}
}
public byte[] GetPackagedEntropy()
{
// Returns a compact byte array that represents almost all of the entropy.
return GetBytes((int)(CurrentEntropyBits / 8));
}
public double GetDouble()
{
// returns a uniformly distributed number on [0-1)
return (double)BitConverter.ToUInt64(GetBytes(8), 0) / ((double)UInt64.MaxValue + 1);
}
public double GetInt(int maxValue)
{
// returns a uniformly distributed integer on [0-maxValue)
return (int)(maxValue * GetDouble());
}
}
class Program
{
static void Main(string[] args)
{
var random = new LowEntropyRandom(2); // this only provides 1 bit of entropy per call
var desiredEntropyBits = 64; // enough for a double
while (true)
{
var adder = new EntropyAccumulator();
while (adder.CurrentEntropyBits < desiredEntropyBits)
{
adder.Add(BitConverter.GetBytes(random.Next()), random.EffectiveEntropyBits);
}
Console.WriteLine(adder.GetDouble());
Console.ReadLine();
}
}
}
}
Since I'm using a 512-bit hash function, that is the max amount of entropy that you can get out of the EntropyAccumulator. This could be fixed, if necessarily.
If I understand your problem correctly, it's that rand() generates finely spaced but ultimately discrete random numbers. And if we multiply it by (y-x) which is large, this spreads these finely spaced floating point values out in a way that is missing many of the floating point values in the range [x,y]. Is that all right?
If so, I think we have a solution already given by Dialecticus. Let me explain why he is right.
First, we know how to generate a random float and then add another floating point value to it. This may produce a round off error due to addition, but it will be in the last decimal place only. Use doubles or something with finer numerical resolution if you want better precision. So, with that caveat, the problem is no harder than finding a random float in the range [0,y-x] with uniform density. Let's say y-x = z. Obviously, since z is a floating point it may not be an integer. We handle the problem in two steps: first we generate the random digits to the left of the decimal point and then generate the random digits to the right of it. Doing both uniformly means their sum is uniformly distributed across the range [0,z] too. Let w be the largest integer <= z. To answer our simplified problem, we can first pick a random integer from the range {0,1,...,w}. Then, step #2 is to add a random float from the unit interval to this random number. This isn't multiplied by any possibly large values, so it has as fine a resolution as the numerical type can have. (Assuming you're using an ideal random floating point number generator.)
So what about the corner case where the random integer was the largest one (i.e. w) and the random float we added to it was larger than z - w so that the random number exceeds the allowed maximum? The answer is simple: do all of it again and check the new result. Repeat until you get a digit in the allowed range. It's an easy proof that a uniformly generated random number which is tossed out and generated again if it's outside an allowed range results in a uniformly generated random in the allowed range. Once you make this key observation, you see that Dialecticus met all your criteria.
When you generate a random number with random(), you get a floating point number between 0 and 1 having an unknown precision (or density, you name it).
And when you multiply it with a number (NUM), you lose this precision, by lg(NUM) (10-based logarithm). So if you multiply by 1000 (NUM=1000), you lose the last 3 digits (lg(1000) = 3).
You may correct this by adding a smaller random number to the original, which has this missing 3 digits. But you don't know the precision, so you can't determine where are they exactly.
I can imagine two scenarios:
(X = range start, Y = range end)
1: you define the precision (PREC, eg. 20 digits, so PREC=20), and consider it enough to generate a random number, so the expression will be:
( random() * (Y-X) + X ) + ( random() / 10 ^ (PREC-trunc(lg(Y-X))) )
with numbers: (X = 500, Y = 1500, PREC = 20)
( random() * (1500-500) + 500 ) + ( random() / 10 ^ (20-trunc(lg(1000))) )
( random() * 1000 + 500 ) + ( random() / 10 ^ (17) )
There are some problems with this:
2 phase random generation (how much will it be random?)
the first random returns 1 -> result can be out of range
2: guess the precision by random numbers
you define some tries (eg. 4) to calculate the precision by generating random numbers and count the precision every time:
- 0.4663164 -> PREC=7
- 0.2581916 -> PREC=7
- 0.9147385 -> PREC=7
- 0.129141 -> PREC=6 -> 7, correcting by the average of the other tries
That's my idea.

"Approximate" greatest common divisor

Suppose you have a list of floating point numbers that are approximately multiples of a common quantity, for example
2.468, 3.700, 6.1699
which are approximately all multiples of 1.234. How would you characterize this "approximate gcd", and how would you proceed to compute or estimate it?
Strictly related to my answer to this question.
You can run Euclid's gcd algorithm with anything smaller then 0.01 (or a small number of your choice) being a pseudo 0. With your numbers:
3.700 = 1 * 2.468 + 1.232,
2.468 = 2 * 1.232 + 0.004.
So the pseudo gcd of the first two numbers is 1.232. Now you take the gcd of this with your last number:
6.1699 = 5 * 1.232 + 0.0099.
So 1.232 is the pseudo gcd, and the mutiples are 2,3,5. To improve this result, you may take the linear regression on the data points:
(2,2.468), (3,3.7), (5,6.1699).
The slope is the improved pseudo gcd.
Caveat: the first part of this is algorithm is numerically unstable - if you start with very dirty data, you are in trouble.
Express your measurements as multiples of the lowest one. Thus your list becomes 1.00000, 1.49919, 2.49996. The fractional parts of these values will be very close to 1/Nths, for some value of N dictated by how close your lowest value is to the fundamental frequency. I would suggest looping through increasing N until you find a sufficiently refined match. In this case, for N=1 (that is, assuming X=2.468 is your fundamental frequency) you would find a standard deviation of 0.3333 (two of the three values are .5 off of X * 1), which is unacceptably high. For N=2 (that is, assuming 2.468/2 is your fundamental frequency) you would find a standard deviation of virtually zero (all three values are within .001 of a multiple of X/2), thus 2.468/2 is your approximate GCD.
The major flaw in my plan is that it works best when the lowest measurement is the most accurate, which is likely not the case. This could be mitigated by performing the entire operation multiple times, discarding the lowest value on the list of measurements each time, then use the list of results of each pass to determine a more precise result. Another way to refine the results would be adjust the GCD to minimize the standard deviation between integer multiples of the GCD and the measured values.
This reminds me of the problem of finding good rational-number approximations of real numbers. The standard technique is a continued-fraction expansion:
def rationalizations(x):
assert 0 <= x
ix = int(x)
yield ix, 1
if x == ix: return
for numer, denom in rationalizations(1.0/(x-ix)):
yield denom + ix * numer, numer
We could apply this directly to Jonathan Leffler's and Sparr's approach:
>>> a, b, c = 2.468, 3.700, 6.1699
>>> b/a, c/a
(1.4991896272285252, 2.4999594813614263)
>>> list(itertools.islice(rationalizations(b/a), 3))
[(1, 1), (3, 2), (925, 617)]
>>> list(itertools.islice(rationalizations(c/a), 3))
[(2, 1), (5, 2), (30847, 12339)]
picking off the first good-enough approximation from each sequence. (3/2 and 5/2 here.) Or instead of directly comparing 3.0/2.0 to 1.499189..., you could notice than 925/617 uses much larger integers than 3/2, making 3/2 an excellent place to stop.
It shouldn't much matter which of the numbers you divide by. (Using a/b and c/b you get 2/3 and 5/3, for instance.) Once you have integer ratios, you could refine the implied estimate of the fundamental using shsmurfy's linear regression. Everybody wins!
I'm assuming all of your numbers are multiples of integer values. For the rest of my explanation, A will denote the "root" frequency you are trying to find and B will be an array of the numbers you have to start with.
What you are trying to do is superficially similar to linear regression. You are trying to find a linear model y=mx+b that minimizes the average distance between a linear model and a set of data. In your case, b=0, m is the root frequency, and y represents the given values. The biggest problem is that the independent variables X are not explicitly given. The only thing we know about X is that all of its members must be integers.
Your first task is trying to determine these independent variables. The best method I can think of at the moment assumes that the given frequencies have nearly consecutive indexes (x_1=x_0+n). So B_0/B_1=(x_0)/(x_0+n) given a (hopefully) small integer n. You can then take advantage of the fact that x_0 = n/(B_1-B_0), start with n=1, and keep ratcheting it up until k-rnd(k) is within a certain threshold. After you have x_0 (the initial index), you can approximate the root frequency (A = B_0/x_0). Then you can approximate the other indexes by finding x_n = rnd(B_n/A). This method is not very robust and will probably fail if the error in the data is large.
If you want a better approximation of the root frequency A, you can use linear regression to minimize the error of the linear model now that you have the corresponding dependent variables. The easiest method to do so uses least squares fitting. Wolfram's Mathworld has a in-depth mathematical treatment of the issue, but a fairly simple explanation can be found with some googling.
Interesting question...not easy.
I suppose I would look at the ratios of the sample values:
3.700 / 2.468 = 1.499...
6.1699 / 2.468 = 2.4999...
6.1699 / 3.700 = 1.6675...
And I'd then be looking for a simple ratio of integers in those results.
1.499 ~= 3/2
2.4999 ~= 5/2
1.6675 ~= 5/3
I haven't chased it through, but somewhere along the line, you decide that an error of 1:1000 or something is good enough, and you back-track to find the base approximate GCD.
The solution which I've seen and used myself is to choose some constant, say 1000, multiply all numbers by this constant, round them to integers, find the GCD of these integers using the standard algorithm and then divide the result by the said constant (1000). The larger the constant, the higher the precision.
This is a reformulaiton of shsmurfy's solution when you a priori choose 3 positive tolerances (e1,e2,e3)
The problem is then to search smallest positive integers (n1,n2,n3) and thus largest root frequency f such that:
f1 = n1*f +/- e1
f2 = n2*f +/- e2
f3 = n3*f +/- e3
We assume 0 <= f1 <= f2 <= f3
If we fix n1, then we get these relations:
f is in interval I1=[(f1-e1)/n1 , (f1+e1)/n1]
n2 is in interval I2=[n1*(f2-e2)/(f1+e1) , n1*(f2+e2)/(f1-e1)]
n3 is in interval I3=[n1*(f3-e3)/(f1+e1) , n1*(f3+e3)/(f1-e1)]
We start with n1 = 1, then increment n1 until the interval I2 and I3 contain an integer - that is floor(I2min) different from floor(I2max) same with I3
We then choose smallest integer n2 in interval I2, and smallest integer n3 in interval I3.
Assuming normal distribution of floating point errors, the most probable estimate of root frequency f is the one minimizing
J = (f1/n1 - f)^2 + (f2/n2 - f)^2 + (f3/n3 - f)^2
That is
f = (f1/n1 + f2/n2 + f3/n3)/3
If there are several integers n2,n3 in intervals I2,I3 we could also choose the pair that minimize the residue
min(J)*3/2=(f1/n1)^2+(f2/n2)^2+(f3/n3)^2-(f1/n1)*(f2/n2)-(f1/n1)*(f3/n3)-(f2/n2)*(f3/n3)
Another variant could be to continue iteration and try to minimize another criterium like min(J(n1))*n1, until f falls below a certain frequency (n1 reaches an upper limit)...
I found this question looking for answers for mine in MathStackExchange (here and here).
I've only managed (yet) to measure the appeal of a fundamental frequency given a list of harmonic frequencies (following the sound/music nomenclature), which can be useful if you have a reduced number of options and is feasible to compute the appeal of each one and then choose the best fit.
C&P from my question in MSE (there the formatting is prettier):
being v the list {v_1, v_2, ..., v_n}, ordered from lower to higher
mean_sin(v, x) = sum(sin(2*pi*v_i/x), for i in {1, ...,n})/n
mean_cos(v, x) = sum(cos(2*pi*v_i/x), for i in {1, ...,n})/n
gcd_appeal(v, x) = 1 - sqrt(mean_sin(v, x)^2 + (mean_cos(v, x) - 1)^2)/2, which yields a number in the interval [0,1].
The goal is to find the x that maximizes the appeal. Here is the (gcd_appeal) graph for your example [2.468, 3.700, 6.1699], where you find that the optimum GCD is at x = 1.2337899957639993
Edit:
You may find handy this JAVA code to calculate the (fuzzy) divisibility (aka gcd_appeal) of a divisor relative to a list of dividends; you can use it to test which of your candidates makes the best divisor. The code looks ugly because I tried to optimize it for performance.
//returns the mean divisibility of dividend/divisor as a value in the range [0 and 1]
// 0 means no divisibility at all
// 1 means full divisibility
public double divisibility(double divisor, double... dividends) {
double n = dividends.length;
double factor = 2.0 / divisor;
double sum_x = -n;
double sum_y = 0.0;
double[] coord = new double[2];
for (double v : dividends) {
coordinates(v * factor, coord);
sum_x += coord[0];
sum_y += coord[1];
}
double err = 1.0 - Math.sqrt(sum_x * sum_x + sum_y * sum_y) / (2.0 * n);
//Might happen due to approximation error
return err >= 0.0 ? err : 0.0;
}
private void coordinates(double x, double[] out) {
//Bhaskara performant approximation to
//out[0] = Math.cos(Math.PI*x);
//out[1] = Math.sin(Math.PI*x);
long cos_int_part = (long) (x + 0.5);
long sin_int_part = (long) x;
double rem = x - cos_int_part;
if (cos_int_part != sin_int_part) {
double common_s = 4.0 * rem;
double cos_rem_s = common_s * rem - 1.0;
double sin_rem_s = cos_rem_s + common_s + 1.0;
out[0] = (((cos_int_part & 1L) * 8L - 4L) * cos_rem_s) / (cos_rem_s + 5.0);
out[1] = (((sin_int_part & 1L) * 8L - 4L) * sin_rem_s) / (sin_rem_s + 5.0);
} else {
double common_s = 4.0 * rem - 4.0;
double sin_rem_s = common_s * rem;
double cos_rem_s = sin_rem_s + common_s + 3.0;
double common_2 = ((cos_int_part & 1L) * 8L - 4L);
out[0] = (common_2 * cos_rem_s) / (cos_rem_s + 5.0);
out[1] = (common_2 * sin_rem_s) / (sin_rem_s + 5.0);
}
}

Resources